LSAT and Law School Admissions Forum

Get expert LSAT preparation and law school admissions advice from PowerScore Test Preparation.

 Administrator
PowerScore Staff
  • PowerScore Staff
  • Posts: 8917
  • Joined: Feb 02, 2011
|
#88144
Complete Question Explanation
(The complete setup for this game can be found here: lsat/viewtopic.php?f=167&p=88139#p88139)

The correct answer choice is (D).

If M is the first stop, then from the first rule L must be the second stop. Accordingly, F and S are the third and fourth stops, not necessarily in that order. With M as the first stop, R can exit at any stop and still satisfy the second rule. Because answer choice (C) does not contain R, answer choice (C) can be eliminated.

From the third rule, J can never be the first stop, which eliminates answer choice (E). V could exit first.
Because F and S are the last two stops in some order, G can exit the van first and still leave room for J to exit at or before the van reaches F. Thus, G can exit first, leaving G, R, and V as three passengers who can exit first. Accordingly, answer choice (D) is correct.

Another way to look at this question is to note that J is the only variable eliminated from exiting the van at the first stop, and the remaining three passengers are bound by rules that are very likely to be easily satisfied if any of those passengers exits first. Thus, answer choice (D) is likely to be true on that basis as well.

Get the most out of your LSAT Prep Plus subscription.

Analyze and track your performance with our Testing and Analytics Package.